petit high school has one thousand two hundred and fifty-two learners.the number of girls is three times more than the number of boys.how many girls are at petit high school​

Answers

Answer 1

Answer:

939 girls

Step-by-step explanation:

Let us start by writing an equation. We can set the number of boys as x, and the number of girls as 3x since the number of girls is three times the number of boys:

Step 1, writing an equation:

[tex]1252=3x+x[/tex]

We will assume in this case, that the number of girls plus the number of boys adds up to the total number of learners at Petit High School

Step 2, solving the equation:

We can simplify the above equation and isolate the term containing x.

[tex]1252=4x\\x=313[/tex]

Step 3, determining values:

We know that we set # of boys as x and we have just solved for x. Therefore, there are 313 boys in the high school. We have set # of girls as 3x, so [tex]3*313=939[/tex] girls in the high school

There are [tex]\fbox{939}[/tex] girls at Petit High School.

I hope this helps! Let me know if you have any questions :)


Related Questions

simplify:- the following fraction

Answers

Answer:  2 & 59/396

Whole part = 2

Fractional part = 59/396

======================================================

Work Shown:

3 & 3/5 = 3 + 3/5

3 & 3/5 = 15/5 + 3/5

3 & 3/5 = 18/5

--------------------

4 & 8/9 = 4 + 8/9

4 & 8/9 = 36/9 + 8/9

4 & 8/9 = 44/9

--------------------

Now apply the rule that 1/(A/B) = B/A. We apply the reciprocal to the fraction A/B to get B/A

That means something like 1/(18/5) becomes 5/18

Also, 1 over (44/9) becomes 9/44

1 over (3/5) becomes 5/3

-------------------

We now have the following:

(5/18) + (9/44) + (5/3)

We'll come back to this later. But first need the LCD, which means we need the LCM of the denominators.

Write out the prime factorization of each denominator

18 = 2*3*344 = 2*2*113 = 1*3

The unique prime factors are: 2, 3, 11

We have 2 occur at most twice, so 2^2 is part of the LCM.We have 3 occur at most twice, so 3^2 is part of the LCMWe have 11 occur at most 1 time, so 11^1 is part of the LCM

The LCM is 2^2*3^3*11^1 = 1188, which is also the LCD

-------------------

We need to get each fraction to have the LCD

5/18 = (5/18)*(66/66) = 330/1188

9/44 = (9/44)*(27/27) = 243/1188

5/3 = (5/3)*(396/396) = 1980/1188

We can now add the fractions like so

(5/18) + (9/44) + (5/3)

(330/1188) + (243/1188) + (1980/1188)

(330+243+1980)/1188

2553/1188

-------------------

We're almost done. We just need to convert to a mixed number, aka mixed fraction

Using a calculator or long division, you should find that

2553/1188 = 2.1489898989899

approximately. Focus just on the stuff to the left of the decimal point. That's the whole part of the answer.

We can then say: 2553-1188*2 = 177

This forms the numerator of the fractional part. The denominator stays at the LCD 1188

That forms the fraction 177/1188. Divide both parts of that fraction by 3 to fully reduce it. You should get to 59/396

-------------------

So that's how we end up with the mixed number 2 & 59/396

Find the size of the angle below

Answers

Answer:

i wish i could tell you

Step-by-step explanation:


If the area is 30cm and the length is 13,23 cm., what will be the width in
cm?​

Answers

Answer:

ok so the standard equation for finding the area of a equation is BH=A

so if the lenth is 1323 and the width is unknown other wise X

we can make an equation

1323X=30

so we divide 30 by 1323 which is

X=0.02267573696 so

The width is about 0.023 cm

Your Welcome

BIFFY OUT!!!

if you give me the correct answer then I will mark as brainliest​

Answers

It's is b cause it is and yeah

Answer:

a

Step-by-step explanation:


-y-22-13




6< X/2-2


I need help solving both

Answers

Answer:    The first one is y≤11 and the second question is x>16 Hope that helps Goodbye!

Step-by-step explanation:  1. Add 22 to both sides.

-13+2

−y≥−13+2

2 Simplify  -13+2−13+2  to  -11−11.

-y\ge -11

−y≥−11

3 Multiply both sides by -1−1.

y\le 11

y≤11

1 Add 22 to both sides.

6+2<\frac{x}{2}

6+2<  

2

x

 

2 Simplify  6+26+2  to  88.

8<\frac{x}{2}

8<  

2

x

 

3 Multiply both sides by 22.

8\times 2<x

8×2<x

4 Simplify  8\times 28×2  to  1616.

16<x

16<x

5 Switch sides.

x>16

x>16

(x+4)+(x-3)+(2x+2)+(x^2+3x+1)

Answers

Answer:

X^2+7x+4

Step-by-step explanation:

find the derivative of the following function, with a positive index (differentiation)
pls help there’s a picture

Answers

Answer:

2nd option

Step-by-step explanation:

Using the power rule

[tex]\frac{d}{dx}[/tex] (a[tex]x^{n}[/tex] ) = na[tex]x^{n-1}[/tex]

Given

f(x) = [tex]\frac{3}{2\sqrt[3]{x} }[/tex] = [tex]\frac{3}{2x^{\frac{1}{3} } }[/tex] = [tex]\frac{3}{2}[/tex] [tex]x^{-\frac{1}{3} }[/tex] , then

f'(x) = - [tex]\frac{1}{3}[/tex] × [tex]\frac{3}{2}[/tex] [tex]x^{-\frac{4}{3} }[/tex]

      = - [tex]\frac{1}{2}[/tex] × [tex]\frac{1}{x^{\frac{4}{3} } }[/tex]

      = - [tex]\frac{1}{2x^{\frac{4}{3} } }[/tex] = - [tex]\frac{1}{2\sqrt[3]{x^4} }[/tex] [ Note there should be a leading negative ]

What are the zeros of the quadratic function
y = 6x2 + 3x - 45

Answers

Answer:

l think

y= x^2

.

.

.

.

..

..

.

.

.

.

Mark Brainliest please

Answer :


x = −3
x = 5/2 = 2.500

what is the 1QR 14,11,8,1,23,20,17,5,19,10,12,22

Answers

Answer:

Step 1: Put the numbers in order.

1, 2, 5, 6, 7, 9, 12, 15, 18, 19, 27.

Step 2: Find the median.

1, 2, 5, 6, 7, 9, 12, 15, 18, 19, 27.

Step 3: Place parentheses around the numbers above and below the median.

Not necessary statistically, but it makes Q1 and Q3 easier to spot.

(1, 2, 5, 6, 7), 9, (12, 15, 18, 19, 27).

Step 4: Find Q1 and Q3

Think of Q1 as a median in the lower half of the data and think of Q3 as a median for the upper half of data.

(1, 2, 5, 6, 7), 9, ( 12, 15, 18, 19, 27). Q1 = 5 and Q3 = 18.

Step 5: Subtract Q1 from Q3 to find the interquartile range.

18 – 5 = 13.

Step-by-step explanation:

What is the maximum value of the objective function, P, with the given constraints? P=25x+45y 4x+y≤16 x+y≤10 x≥0 y≥0

Answers

Answer:

the maximum is 450 and the minimum is 0.

Step-by-step explanation:

We know that:

P(x, y) = 25*x + 45*y

First, is easy to see that as x and y increase, also does the value of P(x, y)

So we just need to find the largest and smallest possible values of these variables. We also can notice that the variable y is being multiplicated by a larger coefficient than x, so we prioritize larger values of y when we can.

We know that:

4x+y ≤ 16

x + y ≤10

x≥0

y≥0

Let's start with the second inequality, let's solve this for y:

y ≤ 10 - x

and from the first one we get:

y ≤ 16 - 4*x

Just to show that maximizing x does not work, let's do it:

from the second one, knowing that the minimum value of y is y = 0

we have that:

0 ≤ 16 - 4*x

Here the maximum value that y can take is x = 1

0 ≤ 16 - 4*1 = 0

So we can have the combination y = 0 and x = 1, when we maximize x (here we can see that we should not maximize x)

in this case we get:

P(1, 0) = 25*1 + 47*0 = 25

let's write again our inequalities:

y ≤ 10 - x

y ≤ 16 - 4*x

If now we take the minimum value of x, x = 0, we get:

y ≤ 10

y ≤ 16

Because the first one is more restrictive, we know that the maximum value that y can take (when x = 0) is y = 10

in this case we get:

P(0, 10) = 25*0 + 45*10 = 450

As expected, here is the actual maximum for the given restrictions.

For the minimum, we just need to take the two lowest possible values of x and y, which are the two given by the equalities on:

x≥0

y≥0

The smallest values are:

x = 0

y = 0

Replacing that in the equation we get:

P(0, 0) = 25*0 + 47*0 = 0

So the maximum is 450 and the minimum is 0. (with the given restrictions)

Can some one help me with the rest of this chapter! I’ll pay them as-well!

Answers

Answers:

a = 475r = -0.5f(3) = 59.4

=======================================================

Explanation:

Any exponential function is of the form y = a*b^x

Comparing that to y = 475*(0.5)^x, we see that a = 475 and b = 0.5

Set b equal to 1+r and solve for r

1+r = 0.5

r = 0.5-1

r = -0.5

The negative r value tells us that the growth rate is -50%, in other words, we have a 50% decay rate. The amount drops by 50% each time x goes up by 1. This is the same as saying the amount cuts in half each time x goes up by 1.

-----------------------

Lastly, plug in x = 3 to find f(3)

f(x) = 475(0.5)^x

f(3) = 475(0.5)^3

f(3) = 59.375

f(3) = 59.4

a man spends ⅕ of his salary on shoes and ⅔ on school fees. what is the fraction of his salary left. pls help asap​

Answers

Answer:

2/15

Step-by-step explanation:

1/5 + 2/3

= 3/15 + 10/15

= 13/15

15/15 - 13/15

= 2/15

1/5 +2/3
=13/15
15/15-13/15
=2/15

Which expression is equivalent to 24x2 – 22x + 5? ​

Answers

Answer:

C: (12x - 5) ( 2x - 1)

Step-by-step explanation:

12 x ( 2x- 1 ) -5  (2x - 1 )

= (12x - 5 ) ( 2x -1)

Hope it helps !

Answer:

12x-5 2x-1

Step-by-step explanation:

This is the correct answer. Answer C

( Answer Honestly Please )
B. Translate the following English phrases to mathematical statements:
6. 3 times a number (x) is greater than 10
7. A number (x) added by 4 is less than or equal to 7
8. 6 is less than a number (x) divided by 6
9. The ratio of 10 and a number (x) is greater than or equal to 5
10.The sum of x and 7 is less than 4

Answers

Answer:

See below.

Step-by-step explanation:

6. 3 times a number (x) is greater than 10

     3x > 10

7. A number (x) added by 4 is less than or equal to 7

    x+4[tex]\leq[/tex]7

8. 6 is less than a number (x) divided by 6

   6 < x/6

9. The ratio of 10 and a number (x) is greater than or equal to 5

      (10/x) [tex]\geq[/tex] 5

10.The sum of x and 7 is less than 4

     x + 7 < 4

PR and SU are parallel lines.


Which angles are alternate interior angles?

Answers

Answer:

Alternate interior angles are on opposite sides of the transversal, and between the parallel lines

Step-by-step explanation:

how do I solve this?

Answers

Answer: I cant see the problem

Step-by-step explanation:

Answer:

Where is the question?

Step-by-step explanation:

what is the value of x in a circle​

Answers

Answer:

30°

Step-by-step explanation:

x = 180° -60°-90° = 30°

The base of an open rectangular box is of length (2x + 5) cm and width x cm.
The area of this base is 58 cm
The height of the open box is (x - 2) cm.
a) Show that 2x + 5x -58 = 0
) Solve the equation 2x + 5x -58 = 0, giving your answers correct
to 2 decimal places
D) Hence calculate the volume of the box, stating the units of your answer.

Answers

Answer:

a) x = 8.29 cm

D) V = 1125.27 cm³

Step-by-step explanation:

a) We can solve the equation as follows:

[tex] 2x + 5x - 58 = 0 [/tex]

[tex] 2x + 5x = 58 [/tex]

[tex] x = \frac{58}{7} = 8.29 [/tex]

D) The volume of the box is given by:

[tex] V = l*w*h [/tex]

Where:

l: is the length

w: is the width

h: is the height

The length of the rectangular box is:

[tex] l = (2x + 5) cm = (2*8.29 + 5) cm = 21.58 cm [/tex]

The width is x = 8.29 cm.

And the height is:

[tex] h = (x - 2) cm = (8.29 - 2) cm = 6.29 cm [/tex]

Hence, the volume is:

[tex] V = l*w*h = (21.58*8.29*6.29) cm^{3} = 1125.27 cm^{3} [/tex]

I hope it helps you!        

8p²-18p² need help from brainliest​

Answers

Answer:

- 10p2

Step-by-step explanation:

p(8p - 18 p)

p x -10p

ans = -10p2

Answer:  -10p²

This is the same as saying -10p^2

============================================

Explanation:

Both terms 8p² and -18p² are like terms.

We combine them in much the same way as 8-18 = -10, just with a p² attached to everything.

So 8p² - 18p²  = (8-18)p² = -10p²

The coefficient -10 is negative because we're subtracting small minus big. If it was the other way around, then we'd get a positive value.

WILL GIVE BRAINLIEST IF CORRECT!!!

Write an equation that represents the line.
Use exact numbers.

Answers

Answer:

y = 3/4 x - 9/2

Step-by-step explanation:

Slope = 3/4

y = mx + b

y = 3/4x + b

0 = 3/4 (6) + b

0 = 18/4 + b

-9/2 = b

y = 3/4 x - 9/2

The plastic straws were placed under the wooden block to

Answers

Answer:

What? Your not even done.

Step-by-step explanation:

Complete the table of values:

Answers

Formula given: [tex]y=x^{2} -x-6[/tex]

x      -3        -2        -1         0         1          2        3

y       6         0       -4         -6       -4        -4        0

when x = -2

[tex]=(-2)^{2} -(-2)-6[/tex]

[tex]=0[/tex]

when x = -1

[tex]=(-1)^{2} -(-1)-6[/tex]

[tex]=-4[/tex]

when x = 1

[tex]=(1)^{2} -(1)-6[/tex]

[tex]=-6[/tex]

when x = 2

[tex]=(2)^{2} -(2)-6[/tex]

[tex]=-4[/tex]

when x = 3

[tex]=(3)^{2} -(3)-6[/tex]

[tex]=0[/tex]

Answer:

-3          -2          -1          0          1          2          3

6            0          -4         -6         -6       -4          0

Step-by-step explanation:

One is given the following function;

[tex]y=x^2-x-6[/tex]

The problem asks one to evaluate the function for certain values, substitute these values in place of the variable (x) and simplify to evaluate the function.

[tex]-2\\\\(-2)^2-(-2)-6\\=4-(-2)-6\\=4+2-6\\=6-6\\=0[/tex]

[tex]-1\\\\(-1)^2-(-1)-6\\=1-(-1)-6\\=1+1-6\\=2-6\\=-4[/tex]

[tex]1\\\\(1)^2-(1)-6\\=1-(1)-6\\=1-1-6\\=0-6\\=-6[/tex]

[tex]2\\\\(2)^2-(2)-6\\=4-(2)-6\\=4-2-6\\=2-6\\=-4[/tex]

[tex]3\\\\(3)^2-(3)-6\\=9-(3)-6\\=9-3-6\\=6-6\\=0[/tex]

Choose the function whose graph is a parabola.

y = |x + 9|
y = 3\sqrt{x}
y = x 2 - 10
y = -2x

Answers

Answer:

y=x2 -10

Step-by-step explanation:

it is because x is squared

PLEASE HELP ME!!! WHOEVER HELPS ME AND GETS IT RIGHT WILL GET BRAINIEST!!!

Answers

Answer: it would be D

Step-by-step explanation: because the x intercept is zero and the equation solved is 6  

By writing each number correct to 1 significant figure, estimate the value of
37.8 x 13.2
28,5 + 22.1
You must show all your working.​

Answers

Answer:

37.8 x 13.2 =4988.6

28,5 + 22.1 = 50.6

14: A popular retailer offered 20% off their store for the entire weekend. Which of the following is the fraction equivalent of 20%?

Answers

Answer:

1/5

Step-by-step explanation:

20/100 or 1/5

Find the missing angle measure

Answers

Answer:

65.5

Step-by-step explanation:

49 + 82 + e = 180°
49 + 82 = 131
180 - 131 = 49°
e = 49°

It costs a homeowner $4.00 to rent a set of hand tools, plus an additional
$2.00 per hour for the rental. The function that models the cost, c, of renting
tools for h hours is c = 2h + 4.
On a piece of paper, graph the function. Then determine which answer choice
matches the graph you drew.

Answers

Answer:

The correct graph answer is A.

Step-by-step explanation:

Let

c ----> the total cost to rent a set of hand tools

h ----> the number of hours for the rental

we know that

The linear equation that represent the problem is equal to

---> this is the equation of the line into slope intercept form

where

the slope m is equal to

m=$2 per hour

b=$2 ---> the y-intercept (the value of c when the value of h is equal to zero)

therefore

The correct graph is A.

B. C. and D. are incorrect

Which decimal is equivalent to 4/3

Answers

Answer:

1.33333333333 repeating

Step-by-step explanation:

Hope this helps

Answer:

1.3333

Step-by-step explanation:

the value of 1/2 ×3/5 is eqaul to​

Answers

Answer:

3/10

Step-by-step explanation:

=1/2 *3/5

= 1*3/2*5

= 3/10

hope it helps

Other Questions
The value of the square root of 42 What is the opposite of smooth? IS LAGGY CORRECT? Geometric stuff and please explain 8. Change the voice of the sentence : A forest fire destroyed the whole suburb. A tetrad is composed of 1. Ano ang Social Media?2.Ano ang kaugnayan ng social media sa akademikong sulatin?3.Ano ang kaugnayan nito sa mundo ng social media?4.Ano ang epekto nito sa paglikha ng akademikong sulatin?pasagot po please. How does socialization help in the overall development of the people and the society as a whole? Imperialismo en Latinoamrica HELP I NEED THIS FOR ENGLISH SO WHAT IS THAT LIKE GANG CALLED IN THE OUTSIDERS I THINK IT HAS TIGER IN IT LIKE TIGER STREET GANG OR SOMETHING I NEED TO KNOWWW Write the equation of the line that contains the points (-2,5) and (6,9). What is the standard form of 14587.25 help please asap please l what is a bank statement Which suffix means to make?-ant-ac-ize-ish How long will it take the cart to to travel 2.8m, starting from rest? At a specific point on the demand curve for backpacks, the elasticity of demand is calculated to be -1.6. a. At that point, we would describe demand as ________.b. If the price of backpacks fell by 10%, the quantity demanded would rise by ________and revenue for the backpack industry would ______ . c. If the price of backpacks rose by 20% the quantity demanded would fall by _______ and revenue for the backpack industry would ________. A bicycle shop owner offers five styles of mountain bikes for $450, $275, $675, $490, and $300. He wants to increase the mean price but keep the median price and range of prices the same. Suggest a new set of prices for the five styles what rhetorical device were the declarations signers using when they used strong, emotional words to describe the grievances against the king. Police response time to an emergency call is the difference between the time the call is first received by the dispatcher and the time a patrol car radios that it has arrived at the scene. Over a long period of time, it has been determined that the police response time has a normal distribution with a mean of 9.6 minutes and a standard deviation of 2.3 minutes. For a randomly received emergency call, find the following probabilities. (For each answer, enter a number. Round your answers to four decimal places.)a. between 5 and 10 minb. less than 5 minc. more than 10 min True or false: according to Executive Order 8802, in 1941 it was the policy of the United States to encourage everyone in the country to help win the war no matter who they were or where they came from